Pharmaceutical Industry 101

This forum made possible through the generous support of SDN members, donors, and sponsors. Thank you.

IndustryPharmD

Here to Help
10+ Year Member
15+ Year Member
Joined
Jul 7, 2007
Messages
795
Reaction score
76
When I just started pharmacy school, I thought all a pharmacist could do in the industry was do research or sales. Since then, I learned that there is literally no limit to the positions a pharmacist could hold within pharmaceutical industry, though most of them have little to nothing to do with being a pharmacist.

I am currently employed in market research. Through the course of my career, I collaborated with almost every department within the company, I know what almost every function does, particularly marketing, market research, clinical development, medical affairs, regulatory affairs, business development and business and competitive intelligence, and have worked in some of those positions as well.

1. What do you enjoy most about your specialty?

To me, being a pharmacist in the industry is like being a kid in a candy store - there are so many opportunities, it is hard to settle on just one. A Pharm.D. can work in clinical research, pharmacovigilance, marketing, new business development, sales, global communications - and the list goes on and on. The good news is, you can change your job function every two-four years until you find something you really enjoy and want to do for the rest of your career. Many companies encourage employees to float from function to function and from department to department. Well, at least the ones that care about you, do.

As far as my actual position... I love the fact that I am in the middle of all things - I get to collaborate with every department, and contribute in a meaningful way to strategic decisions being made. This is an incredibly satisfying feeling - when you analysis becomes the basis for a great discussion, which then leads to a multi-million dollar decision being made. And I love the opportunity to talk to all different kinds of people, which is something I enjoy the most, professionally and personally.

In addition, there are good things to be said for being in a corporate office environment versus on the front lines of pharmacy. I get to sit down for most of my day, and I get to eat a normal lunch. And there are no disgruntled customers ruining your day.

2. Is there anything you dislike about your specialty?

As far as being a pharmacist in the industry... maybe the fact that you may feel like you don't need a PharmD to do what you do. When I first switched to industry, I missed patient contact - that's one of the reasons I still work retail a couple times a month.

For me personally, the biggest frustration comes from feeling that your work comes to nothing because decisions are already made at the top before you even had something to say. That is why choosing the right company/the right position is so important. Also, the downside is that work follows you home, I have been up at 2am working, occasionally... after leaving work at 9pm and going back to work at 7am the next morning.

3. How many years of post-graduate training does your specialty require?

To get into a desired job function right away - a one or two-year fellowship (if available in that area). Otherwise, 3-5 years of job experience are sufficient to at least get the foot in the door. Certain areas - such as sales, pharmacovigilance, some quality assurance positions - will take pharmacists right out of school. From there, it is generally easy to move on to another function. Fellowship is probably the surest and the shortest way to get where you want to be. An MBA or a Bachelor's degree in business may be helpful, depending on a specialty, but is not a prerequisite. It also helps if you have rotated within the company during your senior clerkships, or did a summer internship there. For some positions on the medical side, a residency, especially a specialty residency, can be very helpful.

4. What is a typical schedule like for your specialty? Are the hours/shifts flexible?

It is usually 8-5 or 9-6. No nights or weekends, though depending on the role, occasional late evenings can occur - but it is usually up to you. Most companies allow working from home for a couple days a week, depending on the job function. Many companies also allow flexible hours with certain core hours when employee should be present (such as 10am-3pm, while start of the day can be anywhere between 6 and 10 am, and the end between 3 and 7 pm). But the downside is that being an overtime-exempt corporate employee, in some busy periods you can be at work 8am to 8pm and spend half you weekend working from home.

5. Where do you see your specialty going in five years?

With all the globalization of industry, mergers, restructuring, changing legislation, increased number of generics and accompanying patent litigations, and increased pressure from government and third-party payors, it's certainly a very interesting time to work within pharmaceutical industry. There will always be a place there for pharmacists, though.


I will be happy to answer any questions about
- what roles pharmacists can perform within a pharmaceutical company
- what fellowships/training programs are available (some of them are not advertised anywhere, and I only learned of them through e-mails from colleagues and professional organizations)
- what are prerequisites for an industry fellowship and how does one prepare/apply for one
- and anything else industry-related (assuming it is not proprietary information)

Members don't see this ad.
 
Last edited:
  • Like
Reactions: 2 users
This is excatly why I love Pharmacy is so wide open. My Goal ultimately work for a pharmaceutical company hopefully one of the major ones traveling abroad every pnce in a while or something but working in a wal-greens sounds good too. Do you have to have an MBA? reading that just made me more excited!

No, an MBA is not a requirement for an entry-level position. It can help your career in certain areas, such as marketing, and it can also be helpful in understanding different aspects of you job if you work on the business side - competitive intelligence, market research, etc. - things that include looking at the financial side. My business background is limited to the Accounting 101 and Microeconomics I took in undergrad, and I do feel it is not quite enough sometimes. Pharm.D.s who work in my department either have their MBAs or are working on them. I definitely plan on getting mine in a few years, when I know what kind of an MBA will be right for me - I do not think your average run-of-the-mill MBA will get me where I would like to be. That is also why I personally do not believe in dual Pharm.D./MBA programs - they do not serve any purpose other than adding a credential to the CV and teaching some skills - essentially useless. MBA is more about networking and getting specific knowledge you know you need to advance your career - which is why the best MBA programs do not generally take candidates fresh out of undergrad, and require at least 3-5 years of work experience, preferably in managerial positions.

Just to sweeten the pie, if you are a top employee, cream of the crop, a company might send you into a top MBA program, full-time, on company money, and provide you with a guaranteed dream job upon completion. Some companies have a deal with Ivy League schools where they send a couple people a year for their MBAs. I would love to get such an opportunity, but I don't know whether I will make it - I don't think I am quite the CEO material. :D
 
Members don't see this ad :)
You mentioned you can get into the industry after 3 to 5 yrs of work experience. So, does this mean working at walgreen for 3 to 5 yrs, one could get a chance of working in the industry.

Working for Waglreen's for 3-5 years would position you well for some entry-level positions, such as a sales representatives (though I hear companies are moving away from using pharmacists in the salesforce because pharmacists salaries are too high), or working in safety, maybe in medical information. Completing a drug information residency would be a good option for someone wanting to get into medical information field. Hospital experience would count too, in getting to these entry-level positions. Perhaps doing a lot of research while at school or working in some kind of academic/research environment would open clinical research positions, though hospital experience would help too, retail to a little lesser extent.

What I meant, however, had to do rather with more desirable (well, by some people) and harder to get into areas. Most companies would be reluctant to hire someone to work, for example, in regulatory affairs right after school, or in market analysis. It is easier to transfer into them once you have several years' experience in one of the entry-level positions.

What is pharmacovigilance?

Drug safety. Entry level positions typically involve working in a call center (though most companies staff them with nurses nowadays, but certain areas are more suited for the Pharm.D.s) and then doing some kind of primary data compilation. That's calls from patients and healthcare providers reporting adverse effects from the company's medications or problems with the company's devices. The other major job function for them (though I am not sure whether one can get there right out of school, I believe experience within drug safety is preferred) is analyzing the data. One of the goals is to determine how likely is the adverse event reported to be related to the company's product vs. disease state itself, another concurrent medication, etc. Another is to determine whether a certain side effect is found much more commonly in the general population vs. the study population. Also a major role for that department is working with various government agencies, both within US and abroad, wherever the product is marketed. A pharmaceutical company is requred submit periodic safety reports (I believe it is every 6 months for the first 2 years a drug is on the market and then yearly for several years in the United States, European regulation is a little different), and it is pharmacovigilance people who prepare them. Also pharmacovigilance might support brand teams thinking about phase IV trials, etc. I have little experience with that area.

What are the available fellowships programs including ones not well marketed?

I will make a separate post about a available programs and link it to the FAQ post above.

What are the prequisites to prepare and apply for a fellowship, assuming you are not fortunate to do a summer internship?

Generally, you need to be a pharmacy school graduate, with decent grades and an enthusiasm for working in pharmacy. There are no specific prerequisites in terms of classes you need to take, etc. Many programs require one or more letters of recommendation, but not all. Residency is not required. I will make a separate post about a general timeline and link it to the FAQ post above.
 
Last edited:
So, you think you might want to work for a pharmaceutical company but you aren't quite sure what you want to do there. Sit down and think WHY. That might help narrow down your options - whether a field-based or a corporate office-based position suites your goals, whether you should seek a research-oriented or a business-related position.

Also think of you family circumstances. Would you be willing to relocate? Pharmaceutical industy is generally concentrated on the coasts. Big Pharma - the traditional pharmaceutical powerhouses - is in the East, mainly concentrated in the Philadelphia-New Jersey-New York-Connecticut area. Most notable exceptions are Eli Lilly in Indiana, Procter & Gamble in Ohio, Abbott in Illinois. Biotech companies cluster mainly in California, but again there are plenty of exceptions. There are also field-based positions such as a sales representative or a medical science liaison who cover different parts of the country, so if you are unwilling to relocate, but are willing to spend a significant amont of time on the road, this might be an option.

Something to note also is that salaries generally start significantly lower for pharmacists in the industry, as compared to retail, and even hospital. However, the annual increases are greater, and eventual earning potential is greater.
 
P-1 and P-2 years

Explore all your options. A significant amount of people who think they know exactly what they want to do at the start of pharmacy school change their mind by the time they graduate.

If you are even remotely interested in research, try to work with a faculty member of a project, preferably (but not necessarily) in your area of interest. That would be a chance for you to learn firsthand whether that's something you would like to do, and also it looks good on your resume.

Get involved in the professional extracurriculars right away. Besides being fun (I joined everything I could, and I had a blast), and beefing up your CV for those pesky residency/fellowship applications, it can give you a number of good leads who can turn into letters of recommendations and/or jobs. You never know, if you are doing a fundraiser and ask a pharmaceutical company to provide a dinner and/or a speaker, that person could help you secure a job later.

P-2 year

In the fall, start gathering information about various summer internships available within pharmaceutical companies. There are positions within Pfizer, AstraZeneca, Merck and many other companies. Your school might have a list already prepared, or you could search online by going onto the companies' websites. Do your research early on as many companies have application deadlines in December and January. This would be an opportunity to explore a specific area of interest and could also provide leads for future jobs/fellowships/etc.

P-3 year

When time comes to choose your rotations for next year, try to have a rotation within a pharmaceutical company. Many schools, such as Rutgers University, USP, etc. make it very easy. Other schools do not provide any, but it is always possible to set one up on your own. Contact companies you would like to visit and ask whether they take students on rotations. I know for a fact that Eli Lilly takes students from unaffiliated schools, I am not sure about other companies, but I am sure there are some. Doing the legwork and setting up your own site will pay off, as it will show you have a strong interest in and enthusiams for the industry to the future interviewers.

If possible, try to keep December and January free, since December is the time for the ASHP Midyear Meeting and the time for you to work on your applications, while January is the time most fellowship programs conduct their interviews.

P-4 year

Do your best on every rotation you have. Your preceptor might write you a stellar letter of recommendation to get you through the door. Or their best friend from pharmacy school might be a preceptor for a fellowship program. Or you might love a rotation so much you would change your mind and ask them for a job outright. :D I almost did once, and I never thought beforehand that I would be interested in Indian Health Service.

By the end of October register for the ASHP Midyear Meeting (held annually in early December). This is the one thing everyone interested in any kind of postgraduate training in pharmacy, be it a residency or a fellowship, should do. Not only does it have the residency showcase to explore different residencies, it also has the PPS service. It has three advantages:
- you can request specific interview slots with the programs that interest you
- you can search through the programs listing, and for many of them that was the first time I have ever heard of them. They do not have information posted online, at least not unless you are a specifically looking for these programs, and this would be your only place to learn about them.
- the last and the best, once you register, you can post your own profile, and what type of position you are interested in, and then the programs can contact YOU! The most popular ones, like the Rutgers Fellowship Institute, will not, but many smaller, less known programs will, saving you some time and effort.

Also, if you are specifically interested in one of the programs offered through the Rutgers Institute for Pharmaceutical Industry Fellowships, and you live on East Coast, you should probably attend the Fellowship Information Day, held in November.

By the time you go to the ASHP Midyear meeting, have your Curriculum Vitae prepared. Have at least two people who are good proofreaders go over it. Most likely your school has a human resources professional who helps students with their CVs - use them. It takes more effort to create a good CV than many people realize, and I have seen some abismal products out there.

Also, ask at least three of your preceptors or professors if they would be willing to write you a letter of recommendation. Three is what's required by most programs (some require none or one). Ask them early, and keep in touch with them so they remember. Give them at least three to four weeks to write a letter. Since many fellowships have deadlines in early January (a couple weeks earlier than most residencies!), you may ask them to write your letters before you go to the Midyear. It is hard, however, if you don't know where you will end up applying, so it might be a good idea to wait, too. Depends on how fast you think your LOR writers will be able to get them in, and how sure you are where you will apply, and whether you will apply at all.

After returning from the Midyear, remember to send your thank-you letters to each and every interviewer! It's amazing how many people forget this simple courtesy, and that leads to their downfall, however great a candidate they otherwise are. Also, I am a big fan of writing personalized thank-you letters , as generic ones seem insulting to me. I would recommend this book, Pharmacy Professional's Guide to Resumes, CVs, Interview if you don't know where to start in terms of your CV, cover letters and thank you letters. It is very basic and you should improve on that, but it's a good start. It also helps you to prepare for the interviews. Your school most likely has a copy, and you could buy it. Also you could participate in APhA-ASP's Patient Counseling Competition, and if you place in top 10, you could request it as your prize (you get to choose 2-3 books OR Lexi-Comp for PDA).

Then you go to on-site interview, and wait for the offers, and make your decision, and finally the weight is lifted off your shoulders. :)
 
So excatly does fellowship include and how do you get there. i want to work in the industry because I would like the oppurtunity to work in more of a business setting I tried to pm you but it wont let me.

Sorry, I had my PMs turned off, but I have turned them on, so you should be able to PM me now.

A fellowship is a training program, which, depending on the site, may be rotational in nature or geared towards a specific job function. Rotationals ones can include only two or three different areas, or they may involve changing gears every two-four months. Their main advantage, I believe, is that they prepare you for a position for which you otherwise wouldn't be considered. However, I am in a fellowship position myself, so I am biased in this aspect. Another advantage is that they (especially the rotational ones) allow the fellow to explore different departments and job functions, something a regular employee cannot do. In addition, fellowship generally have a strong mentoring system in place, so you can meet with and learn from people in high positions within the company. Many, but not all, fellowship programs are affiliated with an academic institution, allowing fellows to participate in teaching, learn the academic side of pharmacy (not just teaching, but also administration) and some also allow for a clinical rotation, such as a month or two in a cancer clinic, or on a cardiology floor, etc. if the fellow wants.
 
klebsiella said:
Hi, I am interested in working in industry when I graduate with my PharmD (2010) but am having a hard time finding information on what types of positions are available. I know I do not want to be a MSL, as sales/marketing is not of interest to me, but what else is out there? How much opportunity/upward mobility is available? How do I steer my career to eventually get hired on? Is there an industrial pharmacy association or group?

Thanks for any information, I truly appreciate it!!

The standard search engines and usual websites with pharmacy jobs do not show any industry positions. You would need to go to the companies' websites to learn what is available. There are plenty of options available - medical communication (this includes people in the home office, who write the master file for the drug, on which all the package inserts in all the different countries where it is sold are based), regulatory affairs (working with FDA to submit INDs, NDAs and other required documentation, or, which is more fun to me, checking your own company's ads and detail aids to make sure they comply with the current laws and guidelines), clinical research (which can be entirely paperwork, or you can work very closely with the patients involved in clinical trials - I have spoken to a lady who ran Phase I trials for some twenty years now, and she mostly works with patients and nurses, checking the study drug, making sure blood/urine/whatever they need samples are taken at appropriate time based on supposed pharmacokinetics of the drug), there are medical strategy who look at the whole market and try to come up with a way to position your own products, etc. :) And actually almost any department within a pharmaceutical company will hire a pharmacist (obvious exceptions are accounting, legal, cafeteria, janitorial :D).

As far as upward mobility, the sky is the limit. There are and have been CEOs out there who have pharmacy background - many of them in biotech, as Big Pharma has turned towards business majors lately, but Dr. Ernest Mario was the CEO of Glaxo, Dr. Joseph Williams was the CEO of Warner-Lambert. Some Vice-Presidents are pharmacists, and many of the middle management. It depends on the person's abilities rather than letters after their names.

If you would read the above posts (tentative timeline and answers previously given), it would give you an idea on how to get your foot in the door. If you have a more specific question, please feel free to ask it here.

As far as associations are concerned, there is Pharmaceutical Industry Practitioner Interest Group within APhA (American Pharmacists Association) but that's it as far as I know... People tend to bundle together more based on their job function rather than degree, I think.

Again, after reading the above posts (and bear with me, the list of the positions available will be coming, but I do have a full-time job and a house to run!) if you have anymore questions, just feel free to ask them here. :)
 
TheChemist said:
Thank you for doing this. I've been searching the internet for awhile now trying to find some more information about the pharmaceutical industry. I'm interested in getting involved with regulatory affairs. I like your suggestion about contacting different companies and trying to set up a rotation during my 4th year, since my school my does not have anything like that set up for us. I was wondering if you knew about any industry fellowships that are available for PhamD's? I'm looking for something on the west coast if possible. I know USC has one but I was wonder if you new of any of the big companies having one that is not associated with a University? Thanks again for posting.

You are very welcome. It's a pleasure for me to share information with others about something I love - and I love my job.

Actually, what aspect of the regulatory do you find interesting? I myself did a rotation within promotions & advertisement division, and it was a lot of fun, more interesting (to me, though I know plenty of fellows who disagree) than submitting NDAs. I guess I have a short-term focus. There is a position in regulatory affairs specifically with promotions & advertisement track, offerent jointly through Purdue University, Johnson & Johnson and FDA (two-year program, requires relocation to Indianapolis, New Jersey, and Maryland in quick succession). Then there are positions that focus on the broad scope of regulatory affairs - these are two-year positions with Schering-Plough, Novartis, Roche and Pfizer (all in conjunction with the Rutgers University, so they are in New Jersey/New York area). There is also a one-year position with Eli Lilly, based in Indianapolis (pays A LOT more than the ones listed before it), it is also the only one not affiliated with an academic institution. Also program done by Biogen Idec and MCPHS in Boston has a one-year regulatory component and a one-year drug safety component. There are other, rotational fellowships, that may include regulatory component. Just bear with me, I will post the list of the programs with links (if available) when I have a chance.

As far as West Coast, the only program I am aware of is run by Amgen, but I think they are doing with together with the University of Michigan. Then, as you said, USC offers one... Almost all the programs center around the East Coast, and most of them are run by Big Pharma, while California is mostly home to biotech companies. Maybe other biotech companies will decide to open fellowship positions... Genentech is certainly growing big enough. Problem with biotech is that they are usually small upstarts, and don't have resources, especially the manpower, to run a fellowship program.

Good luck, and if you have any other questions for me, feel free to ask in this thread. :)
 
I'm looking more for the NDA side of regulatory affairs, only because I think it is a more readily available position with the biotech sector on the west coast. I'm interested to hear your opinion on that and if you see PharmD's in this position. Thanks again.

I would say majority of people I know working in regulatory affairs are pharmacists. It is easier to train a pharmacist, who already knows the basics of the drug development process, what an FDA is, some pharmacy law (the law most important for the regulatory affiars is different than the one asked on MPJE, but law class includes the basics of that too). There is a number of fellowship positions available in regulatory affairs - I have listed them above. I have heard from someone who did a rotation with Amgen this past year that Amgen is willing to hire graduating PharmDs right out of schol for regulatory positions, they are so desperate - and their headquarters are in California. Also, if you are interested in working abroad, being a regulatory affairs specialist is something I have seen most requested on European pharmaceutical industry job sites. :) I would say positions dealing with document submission and review are the most common, so you have a good chance at them. If you are interested, definitely try to rotate within regulatory affairs while still at school, to gain a first-hand experience. As I said, the only program I know that's not really school affiliated is Eli Lilly and they have a nicely structured rotation, where you have scheduled meetings with people from other deparments to learn about their job functions (and there are some unusual ones, such as a PR and goverment relations, you wouldn't expect pharmacists in those roles). They have most of their spots go to Butler and Purdue students, but if you send them a letter of interest early (say, October) you have a chance. Your school might offer some too (I don't know where you are, so I can't say). Check the alumni listing too (if you don't have that readily available on the website, alumni association or alumni relations office might be able to help) and see if any of them work in the industry. Don't be shy, call them up and ask if you could rotate with them (I think most schools allow students to set up their own rotations with minimal paperwok - ours required one form from student, and two from preceptor). I think Abbott also takes students for regulatory affairs positions, but I am not sure if that's only local Illinois students - it doesn't hurt to inquire. Many other companies, like BMS and Novartis and Johnson&Johnson take students on rotations, but it's generally limited to Rutgers and USP students.
 
  • Like
Reactions: 1 user
Just wish to start by thanking you so much for putting this information up. I have been trying to find this kind of details for a long time.

You are welcome. The more people know of the programs available, the better talent they would be able to recruit. :)

- Could you give ball park salary ranges for the different fellowship programs?Rutgers, Eli Lilly, J&J/Purdue/FDA

Typical fellowship stipends range between $35-45,000. A few programs (such as Eli Lilly's Visiting Scientist Program and Procter & Gamble Industry Fellowship) pay in the $60-65,000 range.

- What kind of positions are available post regulatory affairs fellowships and what are the salary ranges for those?

The full spectrum of regulatory affairs positions, which are rather large departments in most big companies. Also moving to a different function is possible if desired - and here it is all up to you, your abilities and what is available out there. Salaries all depend on qualifications and experience. I would say starting is somewhere around $70-75,000. It is significantly lower than retail, if that's what you are trying to ask, but the potential for raise is much higher.
 
I am a Biochemistry major graduate, and looking to get involved into business side of pharmaceutical company (not just lab stuffs). What advices do you have for me to find a job that involves with pharmaceutical company?

So you are just a biochemistry major? Are you planning to go to pharmacy school or medical school? Because I really have no advice for biochemistry majors wanting to get into business side of pharmaceutical industry other than get an MBA or proove yourself useful to them in some other way - I have no idea. I mean, I know people with scientific backgrounds, including biochemists and microbiologists working in Business Analytics, but they didn't come there directly, they either took the long and painful route through sales, or through research/science side. I don't know a way to get into business analytics with a non-business degree other than a related post-PharmD fellowship or getting transferred internally from another department, after showing aptitude for that kind of work. Perhaps you can convince them you are a good hire - if you have taken enough business classes for a minor, but I really don't know.
 
DrugRX said:
I am a career changer from computer programming so I am a bit older than my classmates. Doing fellowship for 2 years is too much but if it is necessary, I guess I have no choice.


It is not necessary. In fact, your previous experience working in a corporate environment may be a big advantage as it is. Did you do specifically programming/coding or did you work in a more analytical function? If you have experience as an analyst, having a Pharm.D. you could easily transition into many business analytics roles.[/quote]

What I want to do in drug company is education for sales people and marketing. Also, I want to work in abroad <cut out personal information - IP>.

Oh, I see. SO, is it "education for sales people and marketing people" or "education for salespeople" and "marketing" (because these are two very different roles). If you want to do education for sales reps, you need a more clinical approach. In fact, I would suggest looking at a residency (PGY1). If marketing, I would suggest taking business classes as your electives (if you school allows professional electives to be business classes, mine did) and here either an MBA or a fellowship would be useful. With current downsizing in many roles, it would be difficult for you to vie for a marketing position with an experienced business major.

Working abroad is definitely an option. Though as my supervisor told me when I was interviewing for my position and asked about the same thing "it's the best of the best, the top performers who get to go abroad". My company does two-year assignments, though I know a lady who has done several in a row. I also know someone who works half a year in US, and half a year abroad. It is all possible. I don't know specifically about Korea, but I know a lot of companies are transitioning part of their R&D business into China. I suppose, if you speak the language of the country where you want to go, that would put you at a comparative advantage (though in no way guarantee you will get the assignment). Another option is applying for a job with a Korean company. I have never looked into Asian market myself, but from what I know about Europe, it seems that regulatory consultants with FDA experience are in high demand. I would expect it to be the case in Asia also, though Asian companies tend to find US companies to be their marketing partners in US, rather than build up their own presence (building a sales network from scratch... I don't even want to think of approaching that!).

Also, I plan to get an MBA along the career and try to move up the ladder of corporation if possible.

MBA will be helpful, though I wouldn't think of it as a magic train upwards. The competition is stiff, and it takes time to move up (unless you join a start-up or a fast-growing company (or a subsidiary of a company with current little presence in US, but large abroad, so they have money to pour into US market)). It's also a big commitment. Would you be willing to start working at 6 in the morning and until 11 at night, and all Saturdays, and maybe even some Sundays? And not just "for now, until I get to the certain level," but for the foreseeable future?

How hard/competitive to land a fellowship? I heard residency is quite a bit competitive.

I would say on average some 40-60 people stop by at the PPS suite at ASHP Midyear meeting, some 10-20 apply for each position, and 3-5 get invited to on-site interviews. That's my eyeballing. Residencies are a different thing, because actually the number of applicants is less than two times the number of positions available (if you look at the Match statistics). However, there are popular programs everyone wants to get (and they interview 15-20 people on average, though they usually have more than one spot, the goal for an interviewer is to get 3-5 people per spot, because most good applicants will be offered more than one position, so they need to account for that), and there are programs that no one really wants/knows about, so they end up with 2-3 applicants for their 1-2 spots. When the computer compiles how the programs rank the applicants, and how the applicants rank the programs, and assing an applicant to a program on his list. You can refer to the Match website for more information, they illustrate it better than I could. Then the leftover spots are advertised to the applicants who did not match. So most people do get placed.

How important the GPA is? I have lower than 3.0 as of now. Will having low GPA be a problem?

None of the programs I interviewed for even asked for my transcript, but I guess they get GPA off the CV... In my personal opinion, and when I will conduct interviews this year at ASHP, it is not the most important thing. Of course, if someone has a 2.1 GPA I will talk to them only to be polite, but I will never seriously consider them. If they do sloppy work on their school assignments, how can I trust them with decisions that can affect multimillion dollar products? I would generally prefer at least 3.0. Being a first year, though, you have plenty of time to bring up your GPA. However, when choosing between someone with 4.0 and no extracurriculars to show vs. someone with plenty of leadership experience, great letters of recommendation, and 3.2 GPA I would choose the latter.

How hard is it for me to get hired and relocated to Korea?

I cannot answer that question. I suggest you google "pharmaceutical industry Korea" or "Pharm.D. job Korea" or simply "pharmaceutical industry positions" and see what you can come up with. There are several websites that list positions with pharmaceutical industry outside US, though I have no links, so it would take you as long to find them as it would take me.

What company are you working for?

I will PM you the answer to this question.

Are you usually free on weekend to do some retail work to pay for loan?

Being a fellow, I have to work, otherwise it would be tough to stretch my stipend. However, the pharmacists who earn a pharmacist's salary generally do not work more than a day a month - more to keep up and for some pocket change for a new purse or something, rather than to help with the living expenses.

Is there anything else that you think I need to do other than things you listed in your post such as going annual meeting and doing industry rotation?

Again, it very much depends on your goals and interests, and no one but yourself can answer this question.
 
Members don't see this ad :)
In my case I've been working my way up the research/lab side of industry (biotech, not big pharma) for ~11 years. I'm now going back and applying currently for fall of '08 for my Pharm D., after spending a lot of time talking to folks in the clinical side at work, as well as deciding what I really wanted to pursue in grad school.

Good luck in you application process! Pharmacy is a good career... though I may be biased. :D

My question: I am currently hoping to really focus on clinical research in graduate school, with the eventual goal of moving back into industry after graduation and a possible fellowship. In your opinion and experience, will the years spent working on the research side (currently I'm an Associate Scientist at one of the more prominent biotechs), help me out in terms of moving up the ladder early on, or being more competitive for higher level positions than more relatively entry level clinical research or regulatory affairs work, when I re-enter industry?

I think it would be a definite advantage. You already have experience working on research projects, writing the reports, working in the industry. You also know people who could recommend you, which is a plus. I don't know how much similarity there is in the reports that pre-clinical vs. clinical people do, but I assume a lot of skills are transferable. If you have experience being a project manager, that would be a big advantage too. I just don't have a lot of experience with research side, sorry. Since you are working for a biotech, do you take your own products into clinical trials or do you license them to big pharma? If you do your own trials, I would suggest inviting someone from the clinical trials to a lunch and asking them these questions. For me, the research is "on THAT side of the cafeteria," and I do not work directly with clinical scientists, I am on the business side, not medical. My experience is limited to working with my professors on two research projects - one bench science (working directly with cell culture and equipment), one clinical (restrospective study, working with a database).

Also do you have any thoughts on whether publications play as much of a role, or any role for that matter, in terms of advancement and being considered for positions on the clinical/regulatory side as they do on the research side?

Definitely. If you have a proven track record of completing projects that result in publications in major journals, people would be more likely to trust you with their potential billion-dollar blockbuster. Unless you want to participate in clinical trials in a not-so-scientific role of a trial pharmacist (dispense the trial meds, make sure blood draws are taken at appropriate times according to expected kinetics, etc.). :)
 
Curious as to how easy it is to move up in Pharm industry.

Fancy hearing that from someone with HR experience. :) Just like everywhere else, it depends on your ability to add value to the organization, and on sheer luck. Even most brilliant people don't get advancement if there are no openings... I cannot say I am an expert on the question, though. I have my hopes up, but I am not anywhere near the top. Yet.

Also, are there any opportunities in the South.

GlaxoSmithKline is based in North Carolina. There are a few generic pharma companies based in Florida. Other than that, I don't know anything other than field-based positions (sales res/scientific liaisons/district managers/etc.)

I also am thinking about getting my MD and combining the MD/PharmD with managment experience and degree. I have heard that Pharm companies hire MD to run trials, etc. I would really like to do this option if I get there but I know nothing except bits and pieces about it. Do you happen to know anything on this subject?

I personally cannot see any added value in having both MD and PharmD. What exactly do you mean by "run trials"? If you mean being a lead investigator, then yes, they are MDs. If you mean being the project manager (responsible for making sure everything runs smoothly, figuring out all the logistics, regulatory compliance, etc.) - then they can be MDs, PharmDs, or scientists. I am on the business side of industry myself, so my experience with clinical trials is limited to what my friends who work there tell me. If you have specific questions, I could ask them and relay the answers. For general information, I suggest clicking on Careers portion of several different companies and looking at the requirements for positions that you find interesting.

If I do not make MD, I definitely want to move up to management in Pharm. I really do not like the actual work of a pharmacist but I love the knowledge. The management option was the main reason I decided to go plus of course the money. I am a businessman by training first. The dollars made sense.

Well, you have plenty of options. You can use your management skills anwhere - retail, hospital, industry, insurance, professional associations... I think it would be quite interesting to work for a professional association, actually, lobbying, position statements...there is something about it.

My professors are not much help here on this subject and I have not even mentioned the MD thing for obvious reasons. Although one of my professors has mentioned to me that I should consider MD school because he thinks I would make an excellent doctor.

I know two people who went to medical school after working for a few years as clinical pharmacists in hospitals. They felt it suited their interests and abilities more. One is now a resident, the other an attending. One of my former classmates (working retail for now) is thinking of going to medical school. I would say it's not the best option for advancement within industry, though. It's for people who feel that being a doctor is what they really want to do, in my opinion.
 
ApothRM said:
Sorry to bother you, but i wanted to ask a few quick questions. I'm a p4 and have alot of interest in industry. I have been researching some fellowship programs, mostly rutgers, and was wondering what these programs look for in a candidate.

Different positions require different qualities, of course. A clinical scientist needs a somewhat different skills and personality than someone in marketing. :) The general things I would mention are not too different from what any employer (or pharmacy school, for that matter) look in a candidate. At least, that's what I will be looking for when I will be conducting interviews at Midyear:

- Confidence. If someone keeps their eyes down, stutters, and gives one-word answers, I will not want them.
- Communication skills. If person doesn't quite understand the question implied, it's most likely they will not be a good fit for my position. This may be less important for other positions, but it's a great skill anywhere. The other component, talking, is very important too. If someone uses poor grammar, or too many ehs, or sentence structure simple enough for a three year old, or cannot express their thought adequately, they won't don't cut it.
- Ability to think. Unfortunately, modern education system seems built around stifling this ability, but it is what the real world, the business world, needs the most. Analytical skills are very important. So are logic and common sense. Your grades and ability to memorize endless facts about drugs are not.
- Honesty. Lie about one little minuscule detail on your CV - and I will not have anything to do with you, AND I will tell everyone I know not to deal with you either.
- Manners and Appearance. My pet peeve is lack of basic manners - which includes not just manner of speech and such, but also wearing unpressed suit or shirt. It is rather disrespectful to the interviewer. Strong perfume/cologne is also a no-no. Some interviewers are allergic, I merely want to kick the person out as soon as possible and go get some oxygen. Also, don't forget to ask your interviewers for their business cards, and don't forget to send personalized thank-you letters. I will make a separate post on business communication, because most people send such abysmal coverletters and thank you letters, they make me cry.
- Leadership experience. By that, I don't mean lines in the resume "vice-president of the ASP chapter at my school". I want to hear specific stories of you exercising leadership skills.
- Good team player. Again, I want specific stories. Examples of you working with people you find unpleasant, etc.
- A connection. To me, a good interview should be a conversation, not an interrogation. After all, if that's the person you have to work with closely for a year, you better like them.

That covers the basics.

Also, do u knnow of any search forums i can use to find more fellowships. Or is rutgers all there is?

The best way is to register for PPS at the ASHP Midyear Meeting and search the postings. It will capture the most (though again not all) programs. I will post a list when I have time.
 
IndustryHopeful said:
Thank you for providing wonderful and useful information on this site for pharmacy students like me whoare interested in pursuing Pharma as a career. :) I've found your information and advice to be very helpful with clarifying the expectations, needs, and application process for industry fellowships.

Thank you for your kind words. I think industry is underappreciated as an opportunity for pharmacists in this country, and I hope to help spread the word.

I am truly looking forward to the list of other fellowships available besides Rutgers you will post soon. Its been very difficult searching for fellowships online. It's would be great to learn that there are more fellowship out there; hence, more opportunities! :D

Again, it is coming.

(1) Do fellowship programs tend to look for candidates with experience relevant to their department (i.e. I completed a medical informations internship, but what if I am interested in marketing or drug safety)?

Not necessarily. Just be prepared to answer why aren't you applying for the fellowship in the area you did your internship and/or why didn't you do an internship in your area of interest. I guarantee you these questions will be asked, and you'd rather have good answers. Even "it's all I could find" and "I didn't know what I wanted to do" can be formed into very good answers.

(2) Whats the best way to research more information about a fellowship? Rutgers provides a list of available fellowships, but no details. Any suggestions?

Actually, Rutgers has the full brochures linked, you can click on the links under "Fellowship positions" tab. Nothing outside of an interview will give you more information. I suggest going to the ASHP Midyear and doing informational interviews for all the positions you find interesting. Another option is to contact the contact person listed and ask if there is a current fellow you could talk to.

(3) Do other fellowships (non-Rutgers) follow the a similar application process as Rutgers (i.e. turn in the application after ASHP Midyear)?

Some do, some don't. It depends on the company and the position. Many do go through the Midyear, but how they conduct the rest of the hiring process (what documents they require, what timelines they follow, etc.) differs significantly from company to company.

(4) Do most companies hire their fellow as a full time employee after the completion of his/her fellowship? If so, which companies?

About half the fellows end up working in the same company. The reason is you need a job opening at the time the fellowship ends. Regardless of how wonderful you are, if there are no positions within your area (or no jobs at all) - you would have to go elsewhere. Considering the current situation, with everyone announcing cost cuts and job cuts until 2010-2011, things don't look rosy...
 
Finally, the most requested thing: The list of the postgraduate training programs that prepare for career in industry.

1. Rutgers University Institute for Pharmaceutical Industry Fellowships

I think it is the oldest, and definitely the biggest fellowship program, with about 70 fellows divided between one-year and two-year programs. Its advantage is the large number of alumni and a proven track record. Because it is associated with an academic institution, there are teaching/precepting opportunities. It may be possible to rotate between different departments in many one-year and especially two-year programs.

Most programs are located in New Jersey, or really close to New Jersey in Pennsylvania or New York, with such companies as Novartis, Pfizer, BMS, Roche, Johnson & Johnson and others participating.

Specialties available (varies from year to year):
- Continuing Medical Education

- Medical Information
- Marketing
- Business Intelligence
- Medical Strategy
- Market Research
- Clinical Operations
- Regulatory Affairs
- Drug Safety
- Clinical Pharmacology
- Preclinical research
- Exploratory Clinical Development (Phase I)
- Clinical Research (Phase II to III)
- Project Management
- Managed Markets

2. Eli Lilly Visiting Scientist Program

This is also a large, established program with an average of 5-10 fellows a year. They are located in Indianapolis, and their compensation is significantly higher than most fellowship programs (about $60,000 rather than about $40,000). Rotation between departments is strongly discouraged and there is no teaching component, though precepting may be an option.

Specialties available (again, varies from year to year):
- Compliance and Ethics
- Medical Information
- Regulatory Affairs
- Pharmacoepidemiology
- Health Outcomes
- Non-clinical Drug Development
- Global Medical Communications

3. GlaxoSmithKline Program

Located in North Carolina, small program associated with several universities - UNC, Duke, Thomas Jefferson. Has been around for several years.

Specialties available:
- Health Outcomes, Drug Information, Clinical Research

4. Medical Information/Medical Communications programs, some of which allow for rotations to other departments and/or teaching, are also available through
- Eisai (New Jersey)
- DesignWrite (New Jersey)
- AstraZeneca (Delaware)
- Centocor (Pennsylvania)
- Cubist Pharmaceutical (Massachusetts)
- Ortho Womens Health & Urology (New Jersey)
- Solvay Pharmaceuticals (Georgia)

There was also a program at Scios in California, but considering the severe job cuts they are facing, it may not be available any longer.

5. Clinical Research programs, some of which allow for rotations to other departments and/or teaching, are also available through
- PPD (North Carolina)
- Novartis/SUNY-Buffalo

6. Two-year Drug Safety fellowship is also available through Johnson and Johnson in New Jersey.

7. There are several rotational programs, that have a fellow go through several departments over one or two years:

- Procter and Gamble (Ohio) - also has higher pay than average
- Amgen (California) - has an academic component at U of Michigan
- Biogen Idec (Massachusetts) - brand-new program, just started in 2007
- NovoNordisk (New Jersey)

8. There is also a program at Purdue University, which has two two-year positions, in Medical Information and in Regulatory Affairs. Their unique feature is the association with FDA for the Regulatory Affairs that is focused exclusively on promotions and advertisement review. They also have a half-year academic component and Purdue, and work with Johnson and Johnson and Eli Lilly for the other components.

9. Forest Labs Fellowship
Located in New York, associated with St. John's University. Two tracks are Clinical Development and Regulatory Affairs, both allow for second-year rotations in different departments of interest. Offers an opportunity to precept/teach pharmacy students.

10. Charles River Fellowship in Pharmaceutical Science
Located in Massachusetts, this two-year fellowship is designed to provide PharmDs with additional pharmaceutical sciences knowledge and skills as related to drug discovery and development.

There are also a variety of Drug Information residencies (I have heard that if you are truly interested in Drug Information, then a non-industry residency is actually better, because it prepares you better, and industry will hire you preferentially), master's programs in public heath/health outcomes research, master's in regulatory affairs, and variety of research-intensive masters and Ph.D. programs.

One best place to meet all these programs? ASHP Midyear Meeting. Almost all of them will be represented, including those I have not mentioned because it has been a while since I was actively researching such opportunities. If you register for the PPS service, you gain access to the searchable database of all the programs and can use that. Or just come and browse. If you post your own profile online, the programs may also contact you if they think you are a suitable candidate for them/you may be interested in their program.

Feel free to ask any questions, I have brochures for some of the programs available and may be able to answer them or give you contact information for your questions.
 
First off I would like to thank you for all the help you are giving everyone. Second, this has been one of the most informative (if not the best) place for getting information on pharmaceutical industry.

You are very welcome. I am glad you find it a useful resource. :) Unfortunately, pharmacy schools do not give a lot of information about fields other than clinical pharmacy practice...

I was wondering if it's common for the companies to transfer you over to another country where they have a branch. Maybe you have heard about it or known someone personally who got transferred, any information is helpful. Thank you very much!

I would not say it is common, but it definitely does take place. I know three people in my department who did two year stints abroad. I know one couple who were transferred abroad and liked it so much that they remained there, it is now their sixth or seventh year. However, it is the opportunity for the best and the brightest, the management material. :) Also, it helps if you speak other languages and are culturally aware. I suppose it also varies from company to company, what their policy is. If you have more specific questions, I can try to answer them.
 
Hi - I just ran in to your post and am hoping you can help me. I am a Pharm D who also has an MD. I have always wanted to find a job where I could use both and have tried to find the right connections for an industry job for years now to no avail. I am in the process of doing my neurology reisidency now and really would love to know my options for pharma jobs when I am finished. I love pharma and medicine and I think industry may be more my area. I love the job flexibility and the opportunity to grow.. I just have a feeling this would be right for me. Please give me as much info as you can, I really appreciate it.

I suppose your options would be things like a medical director for a brand (neurology is a good area to be in right now, by the way), medical science liaison (if you want to travel), project coordinator, investigator (if you want more hands-on job)... there are definitely many options to choose from. Just rememer, that in industry, unless you are directly involved in a clinical trial, you will not have patient interaction, and if you are on the business side, you will not use a lot of your clinical knowledge, and what you will use, you will need to view from a very different angle.
 
The people you knew who got sent overseas, were they volunteers or did they specifically picked out candidates? Also what would be a good path to go down if I was also trying to get transferred overseas? Are there specific departments that are more likely to send their employees overseas to work? And are we allowed to stay there permanently? Thank you again, any information you have for me would be useful.

As I had it told to me, it is an opportunity offered to the most promising individuals, with management potential. At least on the business side. I don't know whether scientific side does transfers, and if they do, how that works. As far as departments, marketing and market research are probably your best bet...

Staying there is a rather complex question. I know one person who did. But you need to prove to the company that you are worth keeping there, that you add more value to the company that way. Otherwise, you may have to apply for work visas, apply for a job not as an expat, but simply apply for a local job. It is definitely uncommon, and I do not know any more detail.
 
Hello,
Do you think that if you went to work in the industry and for whatever reason decided it wasn't for you, that it would be easy to shift into retail?
Thank you!

Yes. Retail needs people badly, if you have a Pharm.D. and a license (and most industry pharmacists keep their licenses current - everyone I know in my company does, and we are not even on the scientific side, we are on commercial side), they will take you. As far as work, it really is not too difficult, you are well prepared after graduating from pharmacy school. Besides, as a new hire you will undergo training before working as a pharmacist on your own. It's a good idea to work retail during school and then continue working a weekend a month to keep your skills fresh and for extra income, too. I do it, a lot of my friends do it. Definitely doable.
 
tadalafil06 said:
I am postgraduate pharmacist currently working in retail. I am not happy with the job. Now i am interested in applying to for the pharmceutical industry fellowships. I donot have any internship experience with any industry companies. Do you have any advice or tips which would help me pursue a career in the industry. Does an MBA help? Or is it difficult to get up the ladder?

I would that usefulness of the fellowship is inversely proportional to your years of experience. If you have been out for a few months to a couple of years, it may be a good idea. However, if you have been practicing for several years, it probably will not provide enough value and it would be a significant pay cut.

If you are interested in a career in industry, try applying for entry-level positoins. Look for medical information or safety positions if you are interested in a home-office job, or you can try being a sales rep if you are int something of that nature. Medical liaison position may be good if you have significan clinical experience.

MBA would be helpful if you wanted to work on the commercial side of Pharma. It may or may not be redundant for someone with a Pharm.D. - it's up to you to decide what type of a job you would like to be doing and whether you would bank more on your skills, knowledge and experience of a pharmacist - or try and enter as a business person.

Moving up is depended entirely on your input and value you provide to the company. Networking helps too. Good luck!
 
nextgenpharm said:
Do you have any advice on specific leadership experience that would improve communication skills, and improve chances of landing a fellowship in industry?

Fortunately or not, there is no magic formula "be a president of this organization, and you are guaranteed admission". I would suggest being involved in organization whose goals you share, and where you feel you can make a real contribution. Any leadership position or even participation in any organization is more than just another line in your resume or something to talk about in the interview. It's the skills you develop. For example, many years ago, when I was an elementary school student, and maybe even middle school student, I was rather shy. However, because I was involved in different organizations, I overcame that, and I can approach any random person and talk to them as if we were best friends after a minute or two - because I had to do that a lot while a national officer. You can also have a variety of experiences, and take something away from each of them. I am sure you know all that by now.:)

I'm pretty interested in working with international aid organizations and doing fundraising for these efforts. Also, am a member of several professional organizations within my school. Do you think this is a good direction?

You are currently in pharmacy school, right? There is IPSF, the International Pharmaceutical Students Federation, which would be a good one to try. Here in US it is a part of APhA-ASP activities. Pharmabridge is a program they sponsor, it involves sending textbooks and reference books that have been published within last 5-10 years to students in Third-World countries. There are other activities as well, and fundraising talents are always welcome. I would suggest community care/charity clinics in your area as another good option - they often serve people from all other the world (though most commonly from Latin America), then provide a more hands-on experience, and also look very good on resume.

Being a member of professional organizations is a great start. However, you need to be an active member - you will be asked about what you have been doing for them during interviews. Obtaining a leadership position would be the logical next step - it doesn't have to be President, Historian or philanthropy chair or something like that works just as well - as long as you can proudly say in the end of your tenure: And this year we have done something we have never done before!

Good luck! :)
 
I do not know where to post it so sorry if i am doing something wrong but on a scale of 1 to 10, 1 being the least corrupted and 10 being the most corrupted relatively speaking(comparing it to other corporations) how would u rank the pharmaceutical industry?
No more and no less than any other industry. We are just subject to a lot more regulations and oversight than others.
 
goldenRetriever said:
I am wondering if you can give me a few pointers for the interviews. What are some of the important/relevant questions I can ask my interviewers to find out if I will be a good fit in their programs? (such as work environment, typical day at work, responsibilities, expectations, etc..)

This is an excellent question, and I will be linking the answer to the FAQ post.

One of the resources (and I am sure there are a lot of other good books out there) I personally used and liked when I were in my P4 year, is The Pharmacy Professional's Guide to Resumes, CVs & Interviewing published by APhA. Most pharmacy schools should have them available for students' use.

- it's a fair game to ask what became of the immediate past fellow - did they stay with the company, did they stay in the same job function
- opinions diverge on whether it is appropriate to ask what qualities the preceptors are looking for in an applicant (or what an ideal candidate to fill this position would be like) - I personally think it's OK to ask
- how long the position has existed
- how many fellows are going to be at your company, how many in your department
- ask about expected travel - some positions involve up to 25% travel, some have virtually no travel expectations
- ask about additional training/professional development available, such as pharmacists networking events within company, seminars, etc.
- where does the department fit within company, is there an opportunity to collaborate with other departments (if that's something you want)
- is there an opportunity to rotate in a different area (some positions allow for that, some require it, some absolutely do not allow for that)
- is active involvement in academic component of the fellowship (if the fellowship is affiliated with an academic institution) encouraged?

If I think of more quesitons, I will add them them to this post.

My personal belief is that good interview flows like a conversation, rather than being an interrogation, so I think most questions should flow naturally out of the course of conversation.
 
I have time to update again, though not for a happy reason (home on bedrest).

Even if you aren't at an East Coast school with abundance of industry rotations you can just sign up for, it doesn't mean you cannot get industry experience before graduation. Here is a list of the final-year industry clerkships that are open to candidates from any school that I am aware of:

1. Eli Lilly Advanced Pharmacy Practice Experience Program
Positions available: Global Operations Labeling, Marketing team, Manufacturing, Clinical Research, Global Scientific Information and Communications, Health Outcomes, Medical Information, Medical Liaison, Global Product Safety, Public Policy Planning and Development, US Regulatory Affairs. Normally 4 weeks, only one per student, uncompensated, no housing provided.

2. Procter & Gamble Rotation Program (website is down)
Positions available:

3. To Be Continued
 
A great opportunity to get some industry experience under your belt as early as the summer after your P1 year (though some programs specify that they want more advanced students, and all programs tend to give preference to students who are further in their professional education). They are virtually all paid positions (pay rate comparable to retail pharmacy student pay), and may or may not provide housing.

1. AstraZeneca Pharmacy Internship Program
Positions available: Regulatory Affairs, AstraZeneca Brand Teams, Drug Information, Managed Care, Pharmacy Affairs. For students who have completed at least 2 years of pharmacy school. Housing not provided.

2. Bimark Medical Communications Internship
Positions available: Pharmaceutical Marketing and Management

3. Eisai Drug Information Internship
Positions available: Drug Information.

4. Eli Lilly Pharmacy Internship Program
Positions available: apparently vary from year to year. General search and application process through job search engine.

5. GlaxoSmithKline Summer Internship
Positions available: Not speified. It seems that student at any year at pharmacy school may apply.

6. Hospira Summer Internships
Positions available: apparently vary from year to year. General search and application process through job search engine.

7. Johnson and Johnson Internships
Positions available: apparently vary from year to year. General search and application process through job search engine.

8. Merck Summer Internships
Positions available: apparently vary from year to year. General search and application process through job search engine.

9. Novartis Summer Internship Program
Positions available: apparently vary from year to year. General search and application process through job search engine.

10. Pfizer Summer Internship Program
Positions available: apparently vary from year to year. General search and application process through job search engine.

11. Roche Summer Internship Program
Positions available: apparently vary from year to year. General search and application process through job search engine.

12. Wyeth Summer Internship Program
Positions available: apparently vary from year to year. General search and application process through job search engine.

13. Genentech Internship Program
Positions available: apparently vary from year to year. General search and application process through job search engine.

14. Amgen Internship Program
Positions available: apparently vary from year to year. General search and application process through job search engine.

A great listing of the industry and non-industry internships is available on the Pharmacy Times website.

This listing is to be updated and improved as more information becomes available. Please PM me if you are aware of programs not listed (I posted that I could find easily, I remember seeing more complete listings)
 
modat said:
First off, I wanted to thank you for your post on Pharma Industry. It helped me a TON for the ASHP midyear meetings.

I received a offer for an on-site interview and I don't want to screw it up. My preceptor told me the interview will be several rounds of interviews with different people for 30 minutes each round. So it will be like reliving midyear allllllll over again.

I was wondering if you have any suggestions for me. I am not nervous, I just don't know what to expect. Are the questions going to be the same basic questions they asked during midyear? What type of questions should I ask? And any other suggestions would be GREATLY appreciated.

Congratulations on getting invited onsite! You have successfully cleared your first big hurdle.

This sounds pretty typical, actually - most postdoc programs such as residencies and fellowships (and in fact many Master's/PhD programs) do half-day or full-day, or even two-day interviews (as a friend of mine did, when she applied for a PhD in neuropsychology or something like that).

As far as questions, I really do not know how different people would do things, but I expect the interviews would follow one or more of the standard formats. Could be a conversation, or could be an interrogation (I forget the scientific term for the interviews where they try to intimidate you), they may or may not have a list of questions. And since I don't know what they were asking at Midyear, I cannot really help you. May be more of the same, maybe some even repeating, or may be questions digging deeper, or maybe whatever they did not have time to ask beforehand. I think that questions of the "give me an example of a situation when" are en vogue, but it may just be my company. From my experience when I had a full day worth of interviews to get my current position, I had to repeat the same things over and over a lot since different people asked a lot of the same questions - what is my background, why I want this position, what I would like to do in five years, how do I see my career developing into the future, what is the most important asset I am going to bring into this department, etc.

General suggestions:

- try to get to the interview the night before, if you have that option, that way you can get situated and relax a little. Keep in mind, in winter flights are less predictable and you may be stuck somewhere for a few hours.
- try to get a full night's sleep. You will need to think quickly.
- look good. Wear a nice suit (you are about to become a professional, wear one! be prepared that people interviewing you may be wearning sweaters instead, don't let it unfaze you). Do your hair nicely (so it doesn't fall in your face, doesn't matter if it's nice or not) if you are a woman, shave if you are a man (unless you wear a trimmed beard which is fine, three-day shadow is NOT OK). Be comfortable in what you are wearning - nothing is more distracting than a candidate constantly picking at their clothes. It won't add you confidence either.
- bring copies of your CV just in case - people would usually have them, but still
- bring samples of your work, you may not have a chance to show them, but in case you get asked - it's nice to have them
- remember to ask for business cards if they are not offered to you at the end of the interview - and do send the thank-yous when you get back (do not hand them onsite, that's tacky in my opinion - I had that happen to me, and I felt that I was getting a run-of-the-mill mandatory piece of nothing, rather than a real sincere thank you).
- if you are asked to present on a topic, be prepared, and think it through, and think of how you could make it relevant to the position and company you are applying for
- be sincere. We can feel it.
- be humble (there was this one really annoying guy who thought he was the best candidate ever... he was the only one no one in the group liked :D)
- if you didn't quite understand the question, please do ask the person to repeat or clarify what they meant - if you answer what you think was asked rather than what was asked, it could be a disaster. Asking for clarification (logically justfied, such as "do you mean this, or that, because fact X could make a difference) could play in your favor.
- if you need some time to think about the question, do not just freeze with an absent look, either ask for a moment to gather your thoughts (perfectly OK) or start talking through it (only if that works for you and you think quickly! this is a double-edged sword)
- be nice and polite to receptionists/security/admins (especially admins!) and everyone you encounter. Amazing that some candidates forget that factor.
- do not wear strong perfume/cologne. Most offices are small, and the smell will linger. Or make us gag while you are there. Again, it has happened to me before.
- express enthusiasm, do not make us feel like we have to prod you to get an answer
- do not say disparaging things any pharmaceutical company, maybe the person across the table from you worked for them and still has a lot of friends over there
- if you are interviewing again with the same people who interviewed you at Midyear, remember something about that conversation and tie it back
- do you research, know the company, the products, the challenges it is facing - especially as applicable to the position you are applying for, as you will be asked those questions.

To be updated if I think of something more later.
 
goldenRetriever said:
I am about to send off my letter of intent. I am holding on to it because I know my recommendation letters haven't been sent yet (the recommenders are still writing). So I wanted to read over my letter of intent to make it as perfect as possible.

Can you tell me what format is good, the traditional business block (with 3 bullets highlighting your qualifications) or a personal statement format?

I personally prefer business block format. I have seen plenty of letters of intent that are two-page essays, and they are annoying to read. I would go with a simple format:

- this is what position(s) I am applying for in standard business-speak, such as "I am writing to express my interest in position A, B and C"
- this is WHY I am interested in these positions (show that you know what the company and the position are about)
- this is what I can BRING to the company (your qualifications)
- close with how much you want it and you are looking to hear back soon

I know fellowships receive tons of letters. What would make my letter stand out?... For instance, one of the groups I'm interested in is neuroscience. Is it ok to use a personal anecdote in the letter of intent? Is it too personal?

If you have already had a face-to-face interview, value of letter of intent is significantly diminished compared to you just sending a letter and a CV and hoping to get invited for an interview. Most positions already know whether they want you or now, the letter would be more of a deciding factor if there are several candidates who show themselves to be equally good during the interviewing process. Unless, of course, it is a spectacularly bad letter that can turn off the reader. I had one of those, where I were on the edge about an applicant, and he put so many different things he was interested in, I thought I would like someone more focused on what WE do, and axed him. :) I think personal anecdotes are OK, if they are kept short and sweet and tie in very clearly with the purpose of the letter. Nobody wants to read a page worth of ramblings that lead nowhere. There may be different opinions on that subject, so I would go with your gut instinct. Did you mention something along those lines while interviewing at Midyear? Was the reaction to that positive from your interviewers? Then it is OK to use it. If you got a negative reaction, then probably it is not a good idea.

As for thank you's - I did not leave a thank you card at the Midyear because it seemed like everyone was doing the same thing. I saw the thank-you cards pile up on the information table. So I sent them when I came home from Midyear (via regular US mail). Is a card sufficient, or would a letter format have been better? (I wrote a lot in the card).

When I was younger and more naive, I have read in some article that "everyone loves getting the little envelopes with thank-you cards, even if they know it is just a formality". However, when I really, really wanted a position, I felt I have more to say than fits on a card. So I would type up a letter, fold it into the card, and then handwrite a short one-sentence thank-you in the card proper and mail it like that. That may be an overkill, but I am very verbose. :D I think handwritten cards are perfectly fine.
 
What is job shadowing and what do you do while there? How do I apply for it? Are there any qualifications I need to have before I'm allowed to? How long do I do it?

Do you know of anything I can do to get experience in pharmacy during school breaks or on weekends?

I would suggest you look at areas of pharmacy other than industry to do your shadowing. We do not represent what most pharmacists do in their daily lives. Most people do not sit in an office and stare at a computer for hours at a time, nor do they attend so many meetings. :)

Job shadowing is asking to follow someone around to see what they do. Generally, the best thing to do is ask someone you know well - a friend, a family member, or at least a pharmacist at your local pharmacy your family always goes to. The other venue is to contact a local pharmacy school and ask them to put you in touch with somebody.

It can be either at a hospital or a community setting - I would go with one you find most interesting, since the purpose of this is ultimately to help you decide whether this is the right field for you. It is NOT just a beef-up for a resume. Please don't look at it this way, otherwise I wouldn't want to see you in a pharmacy school. There are no qualifications (except you would have to be at least 18/high school graduate in some states to be allowed behind the counter), and no pre-set length of time (I would suggest at least a day, or equivalent such as a couple days of 2 hours at a time) - it is for YOU to develop a better understanding of the profession, not to check off another item on the admission committee's list.
 
Hi I am foreign pharmacist and I am pursuing MS in Industrial pharmacy in US so can you tell me what is the scope of getting job in Industry in USA .you may have any of your colleague can give the answer ?
waiting for your prompt reply..........................................................

Unfortunately, I am not familiar with MS in Industrial Pharmacy degree. I have a hunch it is something scientific rather than clinical, since graduate degrees are that way, as opposed to professional degree (currently Pharm.D. only). Therefore, since I do not know what skill set you have, I cannot provide an opinion on paths available. I don't even know your interests and aspirations.

One piece of advice I could share would be work on communication skills, both oral and written, since they are used a lot in any role in pharmaceutical industry. That's one of those universal can't-go-wrong advice. :D
 
I have heard that you can enter a pharm school after 2-3 years of college. I'd love to do this if it is possible.

Yes, it is true. Different schools have different minimum entrance requirements, but the minimum is usually 60-65 hours. AP credit and CLEP credit may or may not be accepted, depending on the institution and/or whether the courses are in core areas.

Only thing I am worried about is not getting a bachelors. How will not having a bachelors affect my career? It would be great entering a school early but will it have any negative effects, either long-term say like if I ever wanted to move up. Will it put me at an economic disadvantage, where people may look down on it or it may close some great money opprotunities?

As in, not having another two letters behind your name? First of all, that would not matter beyond your first job out of school. Secondly, never get something just for credential's sake. It's not an approach that will help you advance in your career. If you know you have a specific interest in a certain area, such as business - then certainly a degree in business obtained prior to pharmacy school will be beneficial, as it will provide specific knowledge you will need later in life. Or if your dream is to devote your life to infectious disease, then a bachelor's in microbiology may help you, or a bachelor's in chemistry if your dream job involves more of bench science research. This is the approach I would take when thinking about all this. For now, it looks like you are completely undecided as far as your future career, so I would take the time and explore all options before throwing yourself at something and trying to get into a field too quickly only to regret it later. Fast tracks are for people who are very sure about what they want to do.

Also what is the "ladder" like in the field? Are there advancment opprotunities such as manager, director, chief, ect? Basically what is the ladder like and how difficult is it to move up? I am a hard worker and always strive to do something more. But in my recent research I haven't really seen that many different positions on a "ladder" like scale. Just wondering about that.

There are a couple hundred different functional areas a pharmacist may work in. Therefore, it is not possible to answer your question. Obviously, there would be different opportunities for advancement and different skills sets necessary for a career in retail vs. industry vs. hospital vs. managed care. All I can say, a person who is determined and skilled will always have plenty of opportunity.
 
What is the pay like? :oops:

Silly question, kind of like "What's the cost of bread?" It depends on your education, experience, years at the same company, title, performance, job function, negotiating skills, fringe benefits, etc.

For most the entry-level position, the pay is significantly less than retail starting salary, and usually comparable to hospital. However, there is a much greater opportunity for the compensation to increase...
 
can you work part time at CVS pharmacy or something like that to generate extra income?
If you are licensed - of course, why not? It's purely a personal choice - how much that extra few thousand dollars mean to you, how much your personal time means to you. The great thing is that you can choose to moonlight as much or as little as you want - anywhere from every night and weekend (though I doubt anyone could survive workig 80+ hour weeks for a long time) to one day every three months. It's not just money, it's being able to keep up with the "real world" and also can help the primary job function.
 
I know that for retail positions, getting hired wouldn't really matter where you got your degree but do you think pharmaceutical companies look at say, Maryland or California pharmacy schools as providing a better education (and thus better qualified applicant) than a newer three-year program? Say two people graduated in the top third of their pharmacy class. one from UCSF and one from a lesser-known three year program. Who looks better?

Nobody cares about the school. Pharmaceutical companies want the best person for the job, not the prettiest diploma to decorate an office. The criteria are not too different from any other company, methinks:

a) Does the candidate have the necessary credentials? (can be a certain degree, certificate, etc.)
b) Does the candidate have the skills to do the job?
c) Does the candidate have the "soft skills"?
d) Does the candidate seem like someone pleasant to work with? Do the team members like the candidate?

Additional things that can help sway odds in your favor:
a) Experience in that area
b) Being employed in that company/sister company in a different area and therefore being known as a strong performer to the company
c) Having someone in the company who can recommend you (a very, very important one!)

The school would only matter if someone interviewing you happened to go there/has friends who went there, that helps establish a better connection. In fact, while UCSF is considered by many to be the #1 pharmacy school produced the weakest out of our five recent candidates that I was involved with interviewing. And the best one came from some school in Texas that I never heard of. We are all limited by geography, so it helps you went to a local school - at least people will have heard of it.

The above applied to the candidates who already made the first cut and got invited onsite. The candidates who submit their resumes... there are far too many applicants, and where they went to school is unimportant. I only look at that section if something else catches my eye. And what it is varies from job to job, of course. :) But again, I am not HR, I did hiring for a very specific position. I suppose overall there is a preference for hiring internal candidates and candidates someone internal recommends. It's hard to get into the industry completely from outide. I would suggest completing a fellowship, that would give you a full Rolodex, which is just as important as the specific skills you would develop.
 
hey Industry PharmD , interesting topic , thanks alot for posting it
You are welcome. Industry is not something many US pharmacy students get exposed to, and I try to provide a more or less objective information so they can make more educated decisions about their future careers.

1-since u r in the field . how can u see my odds of getting a Job at a phramceutical company in the US after I graduate (let's say with a GPA of B-)..and what do u think might be increase my chances

Number one question is your visa status. Would you provide enough value for a company to make them go through the trouble of securing a work visa for you? Unless you have some kind of permanent resident visa that would be difficult. Also, no one here knows what pharmaceutical education in Egypt entails. All I remember is what little bits a former classmate of mine who did an exchange to Egypt through IPSF told me, but that was years ago and all I remember is that hospital pharmacists do go on rounds sometimes, though less than in US. I would say a safer bet would be getting hired by an Egyptial subsidiary of a US company and then getting transferred to US (if that's what your ultimate goal is). As far as increasing your chances... unless you have contacts within industry, it would be very, very tough. It is hard to enter industry straight out of school for US pharmacy students. And English not being your native language would place you at a significant disadvantage for most common entry-level positions such as medical information. So another possibility would be getting work experience in another setting, whether in Egypt or US, and then make the transition once you can prove to the company you can add value to them. Also, you need a very good knowledge of US healthcare system and rules and regulations. I don't know how Egyptial system is, and whether you are familiar with US system.

2-would a master in pharmaceutical sciences be a real help to get me into a company

What exactly is it what you want to do? If you want to work in research, yes it would. However, remember that in order to get anywhere you would need a PhD. To give a better advice I would need to know what career path you are interested in and what is it you want to do in industry.

If you have an opportunity to do an internship at a company here in US I would strongly suggest you explore that opportunity. Good luck!
 
-well , u wanted to know which exactly career path I am interested in right now .. I think Quality control is very good but what I like most is Research and development .. So could u give me a better scope on entry requirements for such positions

Quality control is a good entry-level option for pharmacists... again, you would have to do some hard selling to convince people that Egyptian degree prepares you well. I doubt you cand find more than a lot of people who have any familiarity at all with Egyptian education. To make a career in R&D I would suggest getting a Master's or a PhD in pharmaceutical sciences or pharmacology here in US, that would help you. Getting any kind of degree in US always helps when looking for a job in a US company. As far as specific entry requirements, try checking out job postings on the companies' websites. I am strictly on the commerical side and I have never looked into specific job entry requirements on the scientific side.

-and about the master program question .. Do u mean that a MS in pharmacology or Industrial pharmacy is worthless for companies unless I can earn a PhD ?? and would nt it become even an advantage..

Do not attribute to me what I did not say. And please learn to use proper grammar and orthography. Can a Master's degree help you enter industry in US? Yes it can. Can you make a career in R&D without a PhD? NO. If you want a job - you can find a steady paycheck with an MS. If you want a career in R&D - you need a PhD. As simple as that.

I am actually dying to get an internship for a pharmaceutical company in the US .. i am searching from now for One in the summer anywhere between July and October .. Could you give me some tips about how to get one in QC or R&D
Most application deadlines for the programs I know have already closed. You should have started looking in October, November at the latest. Besides, US internships usually run between May and August, since that's when the summer vacation is for most US schools. And as far as QC and R&D - well, research experience in school will give you an edge. Otherwise, same interviewing skills apply as to any other job or program.
 
what would your advice be for someone to go into industrial pharmacy in US after graduating from uk?

Again, what is it what you want to do? The answer is very different whether you want to be a sales representative or work in market research or in clinical research.

Unless you want to be a sales rep, where anyone has a fair shot assuming they have some basic skills, I would advise you to think about what unique qualifications and/or talets can you offer? It is almost universally more difficult to get an entry-level job in another country unless there is a shortage (and there is no shortage of people wanting to get into industry, not to mention it is still being dominated by the business majors - or PhDs on the science side, though pharmacists have been making good inroads). The exact reason US pharmaceutical industry likes pharmacists is our clinical and real-world pharmacy experience we bring to the table. You will not have this advantage. Therefore, you will be competing with everyone else trying to get in - if it is scientific position, do you know enough science to compete with PhD in clinical pharmacology? If it is business - do you know enough business? If it is medical communication - do you know enough about US regulations to compete with US graduates? You can try, but the probability of success would be very low, just as it would be very difficult for me to get a job, say, in Switzerland. Your best options would be either to get to US, get US experience and/or additional education and then make a switch to industry, or start working for a UK subsidiary of a US company - or a British company (and there is no lack of them, Shire, AstraZeneca, GlaxoSmithKline are the major ones that come to mind, I am sure there are others) and then get transferred or apply once you have some experience. The entire healthcare system (and I am reasonably familiar with British system, though not with British pharmacist education) are so different, that it would be very difficult for you to sell yourself to the company. And there is A LOT of competition. If you think you've got something to convince me or people like me to hire you - go ahead. However, everyone who I know who is foreign-born/educated working here is either through a transfer from our foreign branch, or they have obtained another degree - be it PhD, PharmD or MBA - here in the States.
 
  • Like
Reactions: 1 user
I graudated from the Jersey school more that 13 years ago with a BS degree, went right to doing PhD in Biochemistry (never practice) and now a posdoc in molecular biology (not related to drugs). Now, needed to go back to the pharmacy arena because there is no funding and few jobs available. Would like to work in hospital, but would really like to explore the industry to see if there is a niche for a hardcore scientist that happens to have a pharmacy license (actually need to retake NAPLEX (just passed on Wed) and MPJE).

I'm wondering whether companies really someone like me and if so, how to find jobs that fit my strange qualification. Is it better just to post my resume in some search engines or send it directly to companies human resource departments. I live in CT. I really think that you are the perfect person to help me. Thank you very much.

People are generally hired for their skills, not degrees. What is it what you want to do? No one is going to build a unique position just to fit your skills. If you tell me what department you would like to work in, or what your dream job would be like, I may be better able to advise you.

If you are interested in a hospital, if you have a license, getting a hospital job should be reasonably easy depending on the area. If you are willing to go out of the way, many smaller hospitals who pay less and therefore find it hard to attract and keep pharmacists, may be willing to hire someone without work experience. After a couple years, you could then switch into industry - and depending on the length and quality of hospital experience there are variety of positions you could compete for successfully.

Otherwise, since you have no pharmacist work experience, your degree was BPharm and it has been years ago, you probably would be at a severe disadvantage when trying to pursue a pharmacist track into industry right away. If you want to enter industry right now, a better bet would be to exploit your biochemistry background and get a position in research and development, if that is what you want to do. Connecticut-Massachussetts area is a hot spot for biologics research right now - you should be able to find something.

And again, the more information I have about your background and your goals and aspirations - the better quality advice I would be able to offer.
 
nextgenpharm said:
In a previous post, you mentioned that starting salary for industry pharmacists was around 70-75K with a much higher potential for growth than retail. Was just wondering if you could give a ball-park figure for that.

Well, if you grow up to be a CEO, that would be some $2-3 million plus a few million dollars yearly bonus. :D Though it has become rather unusual nowadays to have pharmacists (or any kind of scientifically trained people) as CEOs, it's mostly businesspeople (and a few lawyers), though the tide may be turning... the only thing constant about pharmaceutical industry is change.

Generally, if you do well and move along, you can have increases of some 10-20% every two-three years, if you are moving up the ladder and especially between companies. In retail it's usually the increases at the rate of inflation and there is not much movement or opportunity to increase in pay grade. Retail would probably have you max out at about $150-160,000 total compensation for 40 hrs/week. Depending on what you do, you can do much better in industry. On the other hand, if you aren't exactly ambitious, you could grow at the rate of inflation in industry as well. It's all in the hands of the individual, industry just offers more opportunities, and the increases come with moving around, not sitting in one position.
 
What are some realistic expectations for a person with a PharmD/MBA degree? Like what can a person expect to do in their daily work, what income they will be making, and how long will it take to see the full effects on the PharmD/MBA program? Ultimatly, how will having an MBA affect ones career as a pharmacist?

What do you mean by "realistic expectations"? I mean, expecting someone to give you a senior management position and seven figure salary is obviously unrealistic. You would be looking at a sr. analyst or manager level positions on the commercial side of business, the area depends much more on your interest than on the degree you hold. That also answers the second question. The income depends more on the position than on degree, you may get some small additive effect for additional degree, but do not expect to be paid more just because you have both. As far as full effect - that depends on the quality of the degrees, of course, and how much effort you put into both obtaining them and using them. A degree is just a piece of paper, it can help you get your foot in the door - the rest is ultimately up to you. Please do not think that having both degrees is a magic bullet.

Having an MBA allows for better understanding of the business side (though there are pharmacists without MBA in senior management, as years of experience are even better), and may help you get a faster start on your career, but its main value is as another tool, another skill set to use. Also remember, that there are many different concentrations possible in MBA programs (though I think the joint degrees all offer just general management MBA), so if you are looking for a specific career, you can pick the one best for you. Such as, if you want to own your own pharmacy or small chain - then concentrating in enterpreneurship may help you be more effective, and soften the learning curve. If you want to have a career in industry, then depending on what exactly you want to do, there are many options - but you wouldn't be using your pharmacist education much on the commercial side of pharmaceutical industry.
 
Sevarious said:
1) I would like to begin working in industry and am open to a variety of jobs. I am currently looking into the medical science liaison position as well as positions in medical communications. Are there other entry level positions I would be qualified for? Coordinating clinical trials or dealing with regulatory issues (FDA, patents, etc) are all things I'm also interested in but I'm not sure if these are entry level.

Do you have any real-world work experience? Have you ever worked as a pharmacist? Are you licensed to practice pharmacy in any state? If not, are you planning on getting licensed? Generally, MSL is not a position someone is hired into off the street, it requires either a few years in practice, or working in home office first. MedComm has entry level positions, however, it is recommended that you be licensed to practice pharmacy. Medical writing is another option if you like that kind of stuff. Coordinating clinical trials is very much NOT an entry-level position. Very few people who just have a PharmD (there is a Master's degree in regulatory science, believe it or not, though that is an unusual find, most people just come with experience) join regulatory right off the bat, and if they do, it will be a long time before they deal with the FDA. It's an honor for the senior people. As an entry-level minion you spend days checking materials for regulatory compliance and writing parts of submission packages... Another entry-level position (from which it's not uncommon to move into regulatory) is Drug Safety (start at the call center level, then move up into a more analytical position).

2) How would you suggest I best go about job hunting? I've spent a lot of time in school already and would rather not I'm not do a fellowship. So far, I've been searching monster.com and looking at drug company web sites. Are recruiters and headhunters worth contacting? Are there other job hunting methods I've neglected?

Recruiters can be good or not, depending on who you get. If they don't charge you for their service - why not try them? Company websites are good, but a lot of the time positions are posted internally first, and only if not filled internally they are opened to the outsiders on the general company website. That's where recruiters can come in handy - again, with the caveat of whether they are good and have experience placing people in you area of interest and/or with your background. Actually, posting your resume on monster.com and careerbuilder.com is not a bad idea. Depending on whether the job you are seeking is business or medical/science related, a resume vs. a cv may be better and vice versa. Again, the age-old adage that the best jobs are found through networking holds true. Ask people around you whether they know someone who works in industry and could lead you to a job.

3) What kinds of starting salaries should I be expecting?

Depends on what position you will get. In most cases, pay is related to the position, not the credentials. Basically, starting out you could be at anywhere 80-120+K base, and the more experience you have (read: the more value you can bring to the company), the more you will make. Retail definitely pays more short-term, if that's what you are asking.
 
psurocks said:
i guess my only question to you is: ive read your posts and stuff, but if i want to do a 5 week school rotation at a pharmaceutical company, where do i start? there are so many of them, im so overwhelmed and dont even know where to start...could you help me out with this?

To answer your question, there are two possibilities:

1) Your school offers rotations in pharmaceutical industry through its final year experiential rotations program. In that case, all you need to do is sign up for one following your schools' normal procedure.

2) Your school does not provide an opportunity to rotate in pharma but does allow the student to set up one or more rotations on their own for credit, assuming certain protocol is followed and proper paperwork is filled out by the host company and the school. Most schools do allow that, but you need to check with your office of experiential education first. After that, you look for a rotation in the same way you look for a job.
- online is a great resource
- - first of all I would do a quick and dirty search with key words such as Pharm.D., internship, rotation, pharmaceutical industry, etc. and see what comes up
- - I posted a couple programs in this thread
- - go to the specific companies' websites, to the Careers section, and look their - sometimes they may advertise them but it is not always the case

- personal communication is a great tool, too
- - as your Office of Experiential Rotations whether someone from your school has done an industry rotation in recent years and contact them through alumni network
- - contact the alumni network and see if there is anybody working in pharma; contact that person and ask them if they could help you out
- - ask your professors whether they know someone who works in pharma and could help you out (and most people do)
- - attend a career fair at your school and see if there are any pharma companies recruiting - get your information in, and ask about pharmacy recruitment contacts - and remember to get and keep the recruiter's card, since you will most likely have to contact them yourself

However, there is no guarantee it will work. I regularly get requests from students from my alma mater or friends of friends inquiring about rotations/internships in my company, and all I can do is refer them to the website because my department does not offer any, and I know few people within the groups who do, because I am on the other side of things. And all we offer is either on the website (internships) or contracted with specific schools in the area, with a certain number of positions a year going divided between their rotating students.

Do have your CV ready, because most programs will want to see it rigth off the bat. Write a nice letter explaining why you are interested in doing a rotation within pharmaceutical industry and what you would like to learn from it.

Another thing to remember, do your research and narrow down your choices before you start searching - do you want to see medical communications side of business? marketing? regulatory? clinical sciences? there are many different opportunities, and you are expected to have at least an idea of which one(s) you want. Programs such as rotations and internships are tools to find talent which could be potentially hired later on - and if you do not show the desirable qualities (and independence, self-motivation, ability to find your own path all rank high on the list), why should the company spend time and money on you?
 
forever27 said:
I have another question. As the globalization of pharma industry, how big the potential to work in pharma companies' oversea branches? such as Asian countries? Do they prefer people with relative backgroud?

It's very difficult question. I certainly believe there is a lot of opportunity to work overseas, because that's something I would like to do some day. However, I think it varies greatly from company to company, and I can only speak to the two companies I have been with.

A lot of companies are now expanding into Asia, China and India especially are considered hot markets, so there would definitely be opportunity there. Other growing markets are Latin America and Eastern Europe.

As far as what types of background are preferred... speaking the local language is definitely a plus. And project managers/clinical researchers, marketers and market research people I think are most likely to get to go overseas. Depends on the needs of the specific market. I heard that many areas that are not as desirable (I mean, everyone wants to go to European capitals, not so many are dying to go to Philippines or something like that) need upper-level people willing to move there. At any case, no one will be sending you overseas until you prove yourself, so you best bet would be to start in the industry here in US, and be very upfront about wanting to go to a certain place, and ask people who went there/who have the power to send you there about the best way to accomplish that.
 
gdk420 said:
Question 1:

I was wondering why it is recommend that you have to have your license to work in that position? I thought you don't need a license to work in industry.

Because a lot of companies flat out require that, for one. Whether or not you need a license to work in the pharmaceutical industry very much depends on the position. People on the commercial side do not need it - people in clinical positions must have it, everyone in between - it depends. The reason most med. info positions require a license is that part of your job is answering questions from healthcare providers.

Question 2:

Is Med Comm. the same as Med Info?

Pretty much yes, just different companies name them differently, and sometimes duties are defined a little differently.

Question 3:

What is the difference between clinical pharmacology and early exploratory developement?

I am afraid I am unable to answer this question. I have very little experience with the research side of things.
 
jamoles said:
With regards to industry, what role can a person fulfill if he has earned both degrees? Would it be more legal than pharmacy related? How is this person useful? If you do not know first hand, can you please say why you think he might be useful?

The main position that comes to mind is being a patent attorney. Definitely helps to have both degrees for that position, however, I must warn you that it is a lot less glamorous than people think. It's a ton of memorization and digging through paperwork day to day. Arguing the patent cases is often done by outside counsel...

If this person wanted to work in industry and also at a hospital--say one or two weekends a month--would he need to do a residency and fellowship, or just a residency?

I wouldn't think you need a residency to work in a hospital part-time. At least, I have hard time imagining a hospital giving a part-timer a clinical position (most hospitals do not have them during the weekends anyway), and you certainly don't need additional postgraduate training to staff the IV room or the unit dose. It's a lot more common to moonlight in retail, though, a lot more flexibility that way, too.

THanks so much for your time. Also, as an aside, I cannot tell you how much your thread has helped me see just how diverse a PharmD can be. It has definitely help solidify my decision to pursue pharmacy as a career

You are welcome. :) This was the main reason I chose pharmacy - because there is just so much you can do with this degree, I cannot think of any other that opens so many doors. :)
 
xml2 said:
I will be doing two industry rotations, one in drug safety and one in marketing. Better chances for getting a shot at applying for a fellowship, and also a way for me to get a feel of working for the big pharmas.
Excellent idea, and I am glad you had the opportunity to do more than one - most pharmacy students in US have a hard time trying to find one... Also good that one is on the science side, and one on the business side.

Do you have any specific advice beside doing your best, or arriving early to work just to impress the people whom may write you the recommendations?
I never arrived early, and I still made it through the ranks. :D I did (and still do) stay late, though, sometimes far too late if there is a lot of work. Specific advice that is also general enough for all departments is very hard to come up with, but I will try.

What things you dont want to do? What things you should be doing, and be excellent at?
- unexcused absences are a big no-no
- falling asleep in a meeting is an even bigger no-no
- inappropriate dress/manners (it's a conservative side of business casual in most pharma companies) will irritate people enough that they will not notice even the best work you do
- don't go online to personal interest websites excessively, your internet use can be monitored
- avoid any office politics, just leave the area/get busy with something if you cannot if something of sorts is going on
- do not do an assignment just to have it done. The difference between a student approach and a valuable employee approach to an assignment is that a student takes it at face value and does it exactly as given "I am asked to read this article and give five main points, so here are five main points". What your should do is think WHY is the assignment being given, and WHAT will be done with the result, and approach accordingly. If you remember one thing - remember this, it is the best advice I have ever been given, it has helped me immensely over the years.
- don't EVER think "I am just a student". Ever. Bring up something you feel needs to be talked about. Don't be afraid to take on an active role.
- DO show your enthusiasm for what you are doing, even if it takes all your acting skills
- DO ask relevant questions
- speaking of which, if you have any doubt about your assignment, DO ask questions and clarify it! don't just assume anything
- DO try to meet people from other departments and areas and learn what they do as much as possible
- DO display initiative and offer to take on an additional project if you have time
- if possible, DO ask that you be given your own project that you will be responsible for, rather than help out on a lot of different projects; you will learn a lot more that way, and your value (or lack thereof) would be more apparent
- DO save all records and citations and reference everything you do meticulously; read and memorize the document preservation policy, confidential information notice, etc.
- DO try to connect with your preceptor(s) on a personal level
- if you have a specific personal interest in a certain role or therapeutic area, make it known to your preceptor, they will do their best to accomodate you and give you an opportunity to explore it

Also, pharma companies are not like the ideal world of retail pharmacy in that they will encounter enconomic downturn with possible layoffs that may affect your role in the company. Does that concern you?
Or yes... over 50,000 jobs have been cut in Pharma in the last 15 months or so. Does it concern me? No. If you are a good employee and you know what you are doing, then you will either keep your job, or it will not be too difficult to find another job. There always other positions opening up, even at the time the cuts are taking place. Also, as a pharmacist, you can always go back to retail while you are looking for a real job. Most positions cut now are either sales, or positions supporting failing/expiring products - if you do your research before accepting an offer, you are far less likely to be affected. Also, jobs are being offshored - try to be something that cannot be offshored.
 
forever27 said:
Thanks for your information about fellowship. I am interested in industry pharmacy too. I have a question: what is the difference between the 1-year and 2-years program, regarding the feasibility to locating future job offers, starting salary, program content and competition to get into the program?
As far s the competition to get into the program, it depends on how popular the program (the associated insitution & the specialty area) is that year. Some years research is all the rage - others its marketing, or medical, or regulatory... you never know what you will find until you hear everyone sharing their experiences.

As far as locating future job offers - again, it depends more on what you did your fellowship in and on sheer luck, but maybe a two-year program would offer an advantage, because most good positions require several years of experience, a year in the fellowship accounts for about three. Also, some things, like regulatory, simply require two years for you to even understand what's going on, one year is not adequate. Consequently, your employability in that area would suffer if you only did one year there.

People who did a two-year program should get more money than people who did one year in the same specialty/same position because they have more experience. I cannot judge the magnitude of difference, unfortunately.
 
Hello,

I am having trouble finding this information- What exactly is the difference (besides the advance training of a pharmD) between a pharmacist in industry who wants to work in marketing or sales/ rep vs. a pharmaceutical sales rep? I beleive pharmacist has more flexibility compare to the sales rep as well as better opportunity to move up within the company/ managagement positions.
Thank you for taking the time out to answer this inquiry.
You are mixing several very different positions. The position and the degree are two different things, i.e. a pharmacist can work as a sales rep or work in many other positions. Also, sales and marketing are two completely different functions within an organization.

Therefore, a pharmacist who wants to work as a sales representative, may do so. The same restrictions about speaking on-label only apply, as to anybody else. However, it is very rare to see a pharmacist in a sales rep position (unless they are on a rotation there as part of their development for a specific function within the company). Not only do we cost too much to be sales reps, but also there is a danger that we will speak off-label just because of the knowledge we have.

A pharmacist who wants to work in marketing, generally needs a few years of experience in industry in another position (be it as a sales rep, medical communication, clinical research, or any other position), and business experience and/or MBA may be useful.

The second part of your question leads me to think that you are talking about MSL (medical science liaison, aka scientific liaison, etc. - a field-based position that talks to prescribers and pharmacists much more in-depth than a sales rep ever could, with fewer restrictions). MSLs must have an advanced degree (PharmD is most common, but also MD, PhD, some nurses with doctorate degrees), and their compensation is not related to sales, unlike sales reps.
 
Top